I understand why the answer is A, but I want to understand a few things about answer choice B.
The question stem asks us to choose the answer that does not weaken the resident's argument.
The resident mentions two things--(i) our neighborhood already has the most residents per center of any in the city, and (ii) closing the rec center would be unacceptable, as it would preclude access to rec facilities
Answer Choice B says that children are the main users of rec facilities, and they make up a small portion of the neighborhood's population
This was a tempting answer for me. How exactly does B weaken the argument? How should I have interpreted "main" users and "disproportionately small"?